2021 WSMO Accuracy Round Problems/Problem 7

Revision as of 15:53, 29 January 2022 by Bigkahuna227 (talk | contribs) (Created page with "==Problem 7== Find the value of <math>\sum_{n=1}^{100}\left(\sum_{i=1}^{n}r_i\right),</math> where <math>r_i</math> is the remainder when <math>2^i+3^i</math> is divided by 10...")
(diff) ← Older revision | Latest revision (diff) | Newer revision → (diff)

Problem 7

Find the value of $\sum_{n=1}^{100}\left(\sum_{i=1}^{n}r_i\right),$ where $r_i$ is the remainder when $2^i+3^i$ is divided by 10.

Solution

We will begin by examining $2^i\text{ (mod }10\text{)}$ and $3^i\text{ (mod }10\text{)}$:

\[2^1 \equiv 2\text{ (mod }10\text{)}\] \[2^2 \equiv 4\text{ (mod }10\text{)}\] \[2^3 \equiv 8\text{ (mod }10\text{)}\] \[2^4 \equiv 6\text{ (mod }10\text{)}\] \[...\] and \[3^1 \equiv 3\text{ (mod }10\text{)}\] \[3^2 \equiv 9\text{ (mod }10\text{)}\] \[3^3 \equiv 7\text{ (mod }10\text{)}\] \[3^4 \equiv 1\text{ (mod }10\text{)}\] \[...\]

From this, we can note that:

\[r_i = 5 \text{, } i \equiv 1 \text{ (mod }4\text{)}\] \[r_i = 3 \text{, } i \equiv 2 \text{ (mod }4\text{)}\] \[r_i = 5 \text{, } i \equiv 3 \text{ (mod }4\text{)}\] \[r_i = 7 \text{, } i \equiv 0 \text{ (mod }4\text{)}\]

We can simplify our sum as follows:

\[\sum_{n=1}^{100}\left(\sum_{i=1}^{n}r_i\right)\] \[=100r_1 + 99r_2 + 98r_3 + ... + r_{100}\] \[=(100+96+...+4)r_1 + (99+95+...+3)r_2 + (98+94+...+2)r_3 + (97+93+...+1)r_4\]


Note that $100+96+...+4 = 4(25+24+...+1) = 4(\frac{25 \cdot 26}{2}) = 1300$:

\[=1300r_1 + (1300-25)r_2 + (1300-50)r_3 + (1300-75)r_4\] \[=5(1300) + 3(1275) + 5(1250) + 7(1225)\] \[=\boxed{25150}\]

~BigKahuna227